Đến nội dung

royal1534 nội dung

Có 759 mục bởi royal1534 (Tìm giới hạn từ 20-04-2020)



Sắp theo                Sắp xếp  

#715460 ĐỀ THI CHỌN ĐỘI TUYỂN MÔN TOÁN NĂM HỌC 2018 - 2019 TRƯỜNG THPT CHUYÊN ĐHSP HÀ...

Đã gửi bởi royal1534 on 12-09-2018 - 12:43 trong Thi HSG cấp Tỉnh, Thành phố. Olympic 30-4. Đề thi và kiểm tra đội tuyển các cấp.

 

ĐỀ THI CHỌN ĐỘI TUYỂN MÔN TOÁN NĂM HỌC 2018 - 2019 TRƯỜNG THPT CHUYÊN ĐHSP HÀ NỘI.

Ngày thi thứ hai: 11 - 9 - 2018

Câu 2: Cho các số nguyên $m,n$ lớn hơn $1$ thỏa mãn trong $n$ số $x^{2}-x$ với $x=\overline{1,n}$ không có hai số nào có cùng số dư khi chia cho $m.$ Chứng minh rằng:

(a) $m\geq 2n-1.$

(b) $m=2n-1$ khi và chỉ khi $m$ là số nguyên tố lẻ.

 

Ý b) có thể làm như sau :

Giả sử $m=2n-1$ là hợp số. Đặt $2n-1=xy$ (với $y \geq x>1$). Thì ta phải có $x,y <n$ và $x,y$ lẻ 
Xét đẳng thức sau : $(n-a)^2-(n-a)-(n-b)^2+(n-b)=(b-a)(2n-1-a-b).$
Chọn $a,b$ sao cho $b-a=x;$ $b+a=y$. Hay $b=\frac{x+y}{2};$ $a=\frac{y-x}{2}$. 
Thì suy ra $(b-a)(2n-1+a+b)=x(xy-y)=xy(y-1)$ chia hết cho $xy$ 

Điều này suy ra tồn tại $0\leq a,b \leq n-1$, $a$ khác $b$ sao cho $(n-a)^2-(n-a) \equiv (n-b)^2-(n-b)$ (mod $2n-1$)
Điều này vô lý với đề bài. Vậy $2n-1$ phải là số nguyên tố. 
Từ đây đặt $2n-1=p$ thì dễ kiểm tra được tập các số ${x^2-x}$ với $x=\overline{1,n}$ chứa $n$ số dư khác nhau khi chia cho $p$. 
Vậy $m=2n-1 \Leftrightarrow m$ là số nguyên tố lẻ




#699763 Với mỗi số nguyên dương $m,$ kí hiệu $p(m)$ là ước nguyên...

Đã gửi bởi royal1534 on 05-01-2018 - 16:33 trong Số học

Với mỗi số nguyên dương $m,$ kí hiệu $p(m)$ là ước nguyên tố lớn nhất của $m^{2}+1.$ Cho trước số thực dương $\alpha,$ chứng minh rằng tồn tại vô số số nguyên dương $n$ sao cho: $\frac{p(n)}{n}< \alpha .$ 

Có lẽ đề bài này sai. Chọn $\alpha=1$ thì. Với số tự nhiên $n$ bất kì thì tồn tại một ước $p$ của $n^2+1$ mà $p> \sqrt{n^2+1} >n$. Khi đó $\frac{p(n)}{n}>1$




#698893 TOPIC thảo luận, trao đổi toán thi học sinh giỏi khối 10,11 .

Đã gửi bởi royal1534 on 25-12-2017 - 19:14 trong Chuyên đề toán THPT

Khởi động lạị topic nào

Bài 35.(Sáng tác)

Tìm các hàm: $\mathbb{N}^{*}\rightarrow \mathbb{N}^{*}$ thoả:

$\left\{\begin{matrix}f(1)=2, f(2)=3 & \\ f(f(n))=9f(n)+4n & \end{matrix}\right.$

 

Em xem lại đề bài này, thay $n=1$ thì suy ra $3=22$ vô lý




#698852 Bất đẳng thức với số nguyên dương

Đã gửi bởi royal1534 on 24-12-2017 - 21:45 trong Bất đẳng thức - Cực trị

Cho $a\leq b\leq c$ nguyên dương. Chứng minh $\left \lfloor \frac{c}{a} \right \rfloor+1\geq \left \lfloor \frac{b-1}{a} \right \rfloor+\left \lfloor \frac{c}{b} \right \rfloor$

Bài toán đêm Noel đẹp thật 
Lời giải vắn tất :
Định nghĩa phần nguyên của $ \left \lfloor \frac{x}{y} \right \rfloor$ là thương trong phép chia của $x$ cho $y$ 
+Với $b=a$, bài toán đúng.
+Xét $a+1 \leq b \leq c.$

Đặt $c=bx_{1}+x_{2};b=ay_{1}+y_{2} \Rightarrow c=ax_{1}y_{1}+x_{1}y_{2}+x_{2} $ ($x_{1},y_{1} \geq 1$; $x_{2} \leq b$ , $y_{2} \leq a$) 
Thì $\left \lfloor \frac{c}{b} \right \rfloor=x_{1};\left \lfloor  \frac{b-1}{a}\right \rfloor \leq y_{1}; \left \lfloor \frac{c}{a} \right \rfloor \geq x_{1}y_{1}$
Và ta đưa bài toán về chứng minh $x_{1}y_{1}+1 \geq x_{1}+y_{1} \leftrightarrow (x_{1}-1)(y_{1}-1) \geq 0$




#698403 $x-S(x)$$\geq y-S(y)$

Đã gửi bởi royal1534 on 16-12-2017 - 18:18 trong Số học

Gọi $S(n)$ là tổng các chữ số của số tự nhiên $n$

Chứng minh rằng nếu $x>y$ thì $x-S(x)$$\geq y-S(y)$

Lời giải :

Đặt $x-y=k$

Đpcm $\Leftrightarrow k \geq S(x)-S(x-k)$.

Điều này đúng vì $k \geq S(k) \geq S(x)-S(x-k)$




#698400 TOPIC thảo luận, trao đổi toán thi học sinh giỏi khối 10,11 .

Đã gửi bởi royal1534 on 16-12-2017 - 17:56 trong Chuyên đề toán THPT

Bài toán 34 : Cho $F_{n}=2^{2^n}+1$ là số Fermat thứ $n$.

a, Với mỗi $n$ nguyên dương. Gọi $p$ là ước nguyên tố lẻ bất kì của $F_{n}$. Chứng minh $p \equiv 1$ (mod $2^{n+2}$).

b, Chứng minh tồn tại vô số số nguyên tố $p,q$ phân biệt thỏa $q \mid 2^{p-1}-1$ và $p \mid 2^{q-1}-1$.




#698399 TOPIC thảo luận, trao đổi toán thi học sinh giỏi khối 10,11 .

Đã gửi bởi royal1534 on 16-12-2017 - 17:49 trong Chuyên đề toán THPT

 

Bài 33 (sưu tầm): Tìm n là số nguyên dương, p là số nguyên tố, n $\leq$ 2p sao cho 

 

 $\left ( p-1 \right )^{n}+1 \vdots n^{p-1}$

 

Lời giải : 

Bổ đề (quen thuộc).
Cho $a,n$ nguyên dương. Nếu tồn tại một số nguyên tố $q$ thỏa $q | a^n+1$ và $(q-1,n)=1$ thì $q | a+1$.
Trở lại bài toán : 
Gọi $q$ là ước nguyên tố nhỏ nhất của $n$. Khi đó $(q-1,n)=1$

Suy ra $q | (p-1)+1 \Rightarrow q=1$ hoặc $q=p$ 
Xét $q=1$ thì suy ra $n=1$. Dẫn tới mọi cặp $(n,p)=(1,p)$ đều thỏa mãn
Xét $q=p$.

Suy ra $p^{p-1} \mid n^{p-1} \mid (p-1)^n+1$

Áp dụng bổ đề LTE

$\Rightarrow v_{p}(p-1+1)+v_{p}(n) \geq p-1$

$\Rightarrow v_{p}(n) \geq p-2 $

$\Rightarrow p^{p-2} \mid n$

Mà $n \leq 2p \Rightarrow p=2; 3 \Rightarrow n=2; 3$




#694660 $a^n+2^n \mid b^n+c$

Đã gửi bởi royal1534 on 12-10-2017 - 23:35 trong Số học

Bài toán : Tìm các số nguyên $a,b,c$ $(c \geq 0)$ thỏa $a^{n}+2^{n} \mid b^{n}+c$ với mọi $n$ nguyên dương đồng thời $2ab$ không là số chính phương.




#694053 Chứng minh rằng:a,b đều lẻ

Đã gửi bởi royal1534 on 01-10-2017 - 23:49 trong Số học

Cho 2 số nguyên dương a,b thỏa mãn:$b+1|a^2+1$ và  $a+1|b^2+1$.Chứng minh rằng:a,b đều lẻ

Giả sử $a$ chẵn thì suy ra $b$ chẵn.
Đặt $(b+1,a+1)=d$ . Vì $b+1,a+1$ lẻ nên suy ra $d$ lẻ

$\Rightarrow d \mid a^2-1+b^2-1=a^2+b^2-2$
Mặt khác $d \mid b+1 \mid a^2+1 \Rightarrow d \mid a^2+b^2 $

Từ hai điều trên suy ra $d \mid 2 \Rightarrow d=1$ (vì $d$ lẻ)

Ta có $b+1 \mid a^2+1 \Rightarrow b+1 \mid (a^2+1)+(b^2-1)=a^2+b^2$
Tương tự $a+1 \mid a^2+b^2$

Suy ra $(a+1)(b+1) \mid a^2+b^2$

$\Rightarrow a^2+b^2=n(a+1)(b+1)$ ($n \in Z$)

Sử dụng bước nhảy Viet sẽ suy ra không tồn tại $n$ để phương trình trên có nghiệm.

Từ đó ta phải có $a$ lẻ suy ra $b$ lẻ




#692154 ĐỀ THI HỌC SINH GIỎI LỚP 10 chuyên sư phạm 2016-2017

Đã gửi bởi royal1534 on 02-09-2017 - 21:40 trong Thi HSG cấp Tỉnh, Thành phố. Olympic 30-4. Đề thi và kiểm tra đội tuyển các cấp.

b) Tìm $(a,b,c)$ nguyên sao cho với mọi $p$ nguyên tố lẻ thì:

$a\left ( (\frac{p-1}{2})! \right )^{2}+b\left ( \frac{p-1}{2} \right )!+c\vdots p$

 

 

Lời giải :
Với $p$ là số nguyên tố dạng $4k+3$. Ta có kết quả quen thuộc : $[(\frac{p-1}{2})!]^2 \equiv 1$ (mod p)
Đặt $x=(\frac{p-1}{2})!$. Ta có $x \equiv +1,-1$ (mod p)
Vì $p \mid ax^2+bx+c$. Suy ra $p \mid a+b+c$ hoặc $p \mid a-b+c$. Với mọi $p$ là số nguyên tố dạng $4k+3$.
Suy ra $a+b+c=0$ hoặc $a-b+c=0 $
Nếu $a+b+c=0$.
Ta có $ax^2+bx+c=ax^2-(a+c)x+c=ax(x-1)-c(x-1)=(ax-c)(x-1)$
Cho $p$ là số nguyên tố dạng $4k+1$. Thì ta có $x^2 \equiv -1 (mod p)$. Suy ra $x-1$ không chia hết cho $p$
Vì $p \mid ax^2+bx+c$
$\Rightarrow p \mid (ax-c)(x-1)$
$\Rightarrow p \mid ax-c$
$\Rightarrow p \mid cx+a$
$\Rightarrow p \mid a(cx+a)-c(ax-c)=a^2+c^2 $
$\Rightarrow a^2+c^2=0$
$\Rightarrow a=b=c=0$
Nếu $a-b+c=0$ làm tương tự ta cũng có $a=b=c=0$
Tóm lại $(a,b,c)=(0,0,0)$ là bộ số duy nhất thỏa đề




#680355 Thảo luận về việc làm ĐHV

Đã gửi bởi royal1534 on 12-05-2017 - 01:20 trong Góp ý cho diễn đàn

Vậy sao cậu không vào xem các bài viết của NHoang1608, Mr Cooper, Minhnks, HoangKhanh2002 thử xem. Các bạn đó toàn bài hay hơn, mà Mr Cooper thường đăng bài rất khó, hay, đã đăng kí 2 lần mà ko ai ngó ngàng tới, HoangKhanh2002 đăng kí tận 3 lần mà cũng chưa được là sao, còn việc bạn bảo hồi xưa thì khác giờ rất nhiều, hồi ấy còn ít thành viên và trình độ còn thấp, nên có thể...

Mình không biết bạn là ai và bạn có thật sự khai đúng tuổi của mình như trên nickname hay không nhưng cái suy nghĩ của bạn đúng là nông cạn :) 




#678808 $(a^2+b)(b^2+a)$

Đã gửi bởi royal1534 on 27-04-2017 - 22:59 trong Số học

Tìm số nguyên dương a;b để $(a^2+b)(b^2+a)$là lũy thừa của 2

Giả sử $(a^2+b)(b^2+a)=2^m (m \in Z^{+})$

Đặt $a^2+b=2^x,b^2+a=2^y (x+y=m)$

Không mất tính tổng quát. Giả sử $x \geq y $

$\Rightarrow 2^y \mid 2^x$

$\Rightarrow b^2+a \mid a^2+b$

$\Rightarrow b^2+a \mid a^4-b^2+b^2+a$

$\Rightarrow b^2+a \mid a^4+a$

$\Rightarrow 2^y \mid a(a+1)(a^2-a+1)$

Vì $a^2-a+1=a(a-1)+1$ là số lẻ nên $2^y \mid a(a+1)$

$\Rightarrow 2^y \mid a$ hoặc $2^y \mid a+1$

Suy ra $a \geq b^2+a-1 \Rightarrow 1 \geq b^2 \Rightarrow b=1$

Từ đó tìm được $(a,b)=(1,1)$




#673256 $(a+b)^2(b+c)^2(a+c)^2\geq abc(a+2b+c)(a+2c+b)(2a+b+c)$

Đã gửi bởi royal1534 on 02-03-2017 - 20:22 trong Bất đẳng thức - Cực trị

CRUX

$(a+b)^2(b+c)^2(a+c)^2\geq abc(a+2b+c)(a+2c+b)(2a+b+c)$

Một lời giải khác : 
Đặt $a+b=x,b+c=y,a+c=z$
Ta quy bài toán về chứng minh 
$8x^2y^2z^2 \geq (x+y-z)(z+x-y)(y+z-x)(x+y)(y+z)(z+x)$
Ta dễ chứng minh bất đẳng thức phụ sau (chỉ cần biến đổi tương đương) : 
$\frac{(x+y+z)(x^2+y^2+z^2)}{9} \geq \frac{(x+y)(y+z)(z+x)}{8}$  
Ta quy bài toán về chứng minh: 
$9x^2y^2z^2 \geq (x+y-z)(z+x-y)(y+z-x)(x+y+z)(x^2+y^2+z^2)$ 
$\Leftrightarrow \frac{9x^2y^2z^2}{x^2+y^2+z^2} \geq (x+y-z)(x+z-y)(y+z-x)(x+y+z)$
$\Leftrightarrow \frac{9x^2y^2z^2}{x^2+y^2+z^2} \geq 2(x^2y^2+y^2z^2+z^2x^2)-(x^4+y^4+z^4)$ (Đúng theo BĐT Schur)
Ta có điều phải chứng minh




#671805 $p+p^2+p^3+p^4=q!$

Đã gửi bởi royal1534 on 16-02-2017 - 18:27 trong Số học

Tìm các số nguyên tố $p,q$ sao cho: $p+p^2+p^3+p^4=q!$

Lời giải : 

Phương trình đã cho tương đương với : $p(p+1)(p^2+1)=q!$

Ta có $p \mid q! \Rightarrow p-1<p \leq q \Rightarrow q! >(p-1)!$

Dễ kiểm tra $(p-1)! >p(p+1)(p^2+1)$ với $p \geq 8$

Vậy ta chỉ cần xét trong trường hợp $p \leq 7$




#670307 $x^{2}+y^{2}=n$

Đã gửi bởi royal1534 on 29-01-2017 - 10:14 trong Số học

Tại sao bác lại chọn được $n=3^{2^k}-1$?

À đây là một bài toán mình đọc được trong cuốn Problem From The Book.

Đề bài : Chứng minh rằng phương trình : $3^k=m^2+n^2+1$ có vô số nghiệm nguyên dương.

Đáp án là $k=2^n$ đã gợi ra cho mình cách chọn trên. Và có thể thấy nếu thay đổi số 3 thành một số thích hợp khác thì lời giải không bị thay đổi nhiều




#670239 $a(a+b)(a^2+b^2)+b(b+c)(b^2+c^2)+c(c+d)(c^2+d^2)+d(d+a)(d^2+a^2) \g...

Đã gửi bởi royal1534 on 28-01-2017 - 17:19 trong Bất đẳng thức và cực trị

Bài toán sau là đề ra kì này của tạp chi PI của bạn tháng 11/2016 và đã được đưa giải trong số đầu của tạp chí PI. Tác giả bài toán là thầy Nguyễn Đức Tấn. Mình post bài này để các bạn thảo luận cùng tìm các cách khác nhau để giải. Số 1 của PI đã đưa ra hai lời giải cho bài này.

 

P1. (Nguyễn Đức Tấn - PI số 1) Chứng minh rằng với mọi số thực $a,b,c,d$ ta có bất đẳng thức

$$a(a+b)(a^2+b^2)+b(b+c)(b^2+c^2)+c(c+d)(c^2+d^2)+d(d+a)(d^2+a^2) \ge 0$$

Đầu năm làm bài trong tạp chí Pi cho may mắn :D 

Lời giải :

Đặt $$A=a(a+b)(a^2+b^2)+b(b+c)(b^2+c^2)+c(c+d)(c^2+d^2)+d(d+a)(d^2+a^2)$$

       $$B=b(a+b)(a^2+b^2)+c(b+c)(b^2+c^2)+d(c+d)(c^2+d^2)+a(a+d)(a^2+d^2)$$

Ta có $$A-B=\sum ((a+b)(a^2+b^2)(a-b))=\sum (a^4-b^4)=0 \Rightarrow A=B$$

Vậy thì ta có $$2A=\sum (a^2+b^2)(a+b)^2 \geq 0 \Rightarrow A \geq 0$$




#670223 $x^{2}+y^{2}=n$

Đã gửi bởi royal1534 on 28-01-2017 - 13:46 trong Số học

Tìm $n$ tức là tìm tất cả $n$ để cho nó biểu diễn được không phải chỉ ra tồn tại vô hạn

Đề bài đâu yêu cầu tìm tất cả $n$ đâu anh.

Lời giải trên em chỉ ra với $n=3^{2^k}-1$ hoặc $n$ là số nguyên tố dạng $4k+1$ thì luôn tồn tại $x,y$. (Khi đó chọn $k$ bất kì thì luôn biểu diễn được ._.)

Có gì sai sót đại ca chỉ giáo :D 




#670222 CMR: $\frac{18a^3+20abc}{b+c}\ge 4b^2+4c^2...

Đã gửi bởi royal1534 on 28-01-2017 - 13:40 trong Bất đẳng thức - Cực trị

Lời giải của em hay, nhưng chưa chặt lắm. Ta chỉ có: $a^3+abc\ge a^2(b+c)$ khi $a\ge 0$. Còn $a<0?$

Hic. Tạm thời em sẽ để lời giải với điều kiện $a,b,c$ dương vậy.




#670219 $x^{2}+y^{2}=n$

Đã gửi bởi royal1534 on 28-01-2017 - 13:30 trong Số học

Tìm $n$ nguyên dương thỏa mãn phương trình sau:

$x^{2}+y^{2}=n$

có nghiệm nguyên dương

Mình có 2 cách cho bài toán này.

Cách 1 : Chọn $n$ thuộc tập hợp số nguyên tố dạng $4k+1$. Khi đó theo định lý Fermat-Euler về tổng hai bình phương thì luôn tìm được $x,y$ thỏa mãn phương trình trên.

Cách 2 : Chọn $n=3^{2^k}-1$

Ta đi chứng minh phương trình : $3^{2^k}=x^2+y^2+1$ luôn có nghiệm nguyên dương.

Với $k=1$. Chọn $x=y=2$

Giả sử khẳng định trên đúng đến $k$. Ta chứng minh nó đúng với $k+1$.

Thật vậy. Ta có $3^{2^{k+1}}-1=(3^{2^{k}}-1)(3^{2^{k}}+1)$

Ta có $3^{2^k}-1$ có thể viết dưới dạng tổng hai bình phương (Theo giả thiết quy nạp). Và $3^{2^{k}}+1$ cũng viết được dưới dạng tổng hai bình phương.

Từ đó theo đồng nhất thức : $(a^2+b^2)(c^2+d^2)=(ab+cd)^2+(ac-bd)^2$. Ta suy ra $3^{2^{k+1}}-1$ viết được dưới dạng tổng hai bình phương. Tức là tồn tại $x_{1},y_{1}$ thỏa $3^{2^{k+1}}-1=x_{1}^{2}+y_{1}^{2}$ 

Theo nguyên lý quy nạp. Ta có phương trình $3^{2^k}=x^2+y^2+1$ luôn có nghiệm nguyên dương




#670217 CMR: $\frac{18a^3+20abc}{b+c}\ge 4b^2+4c^2...

Đã gửi bởi royal1534 on 28-01-2017 - 12:57 trong Bất đẳng thức - Cực trị

Cho $a,b,c$ là các số thực thỏa mãn: $\left\{\begin{matrix} a=max(a,b,c)\\b^2(17a+10c)+c^2(17a+10b)=27a^2(b+c) \end{matrix}\right.$.
Chứng minh rằng: $18a^3+20abc\ge (4b^2+4c^2+11bc)(b+c)$

Lời giải (Với điều kiện $a,b,c$ dương) :

Từ giả thiết ta có : $(a-b)(a-c) \geq 0 \Rightarrow a^2+bc \geq ab+ac \Rightarrow a^3+abc \geq a^2(b+c)$

Khai thác giả thiết ta có :

$$17a(b^2+c^2)+10bc(b+c)=27a^2(b+c)$$

$$\Rightarrow 17a(b^2+c^2)+10bc(b+c) \leq 27(a^3+abc)$$

$$\Rightarrow a[17(b^2+c^2)+3bc]+10bc(b+c) \leq 27a^3+30abc$$

Ta có BĐT phụ sau : $17(b^2+c^2)+3bc \geq \frac{37}{4}(b+c)^2 \Leftrightarrow 31(b-c)^2 \geq 0$

$$\Rightarrow 27a^3+30abc \geq \frac{37a}{4}(b+c)^2+10bc(b+c)$$

$$\Rightarrow 18a^3+20abc \geq \frac{2}{3}.[\frac{37a}{4}(b+c)^2+10bc(b+c)]$$

Ta quy bài toán về chứng minh : 

$$\frac{2}{3}.[\frac{37a}{4}(b+c)^2+10bc(b+c)] \geq (b+c)(4b^2+4c^2+11bc)$$

$$\Leftrightarrow \frac{37a(b+c)}{4}+10bc \geq 6(b^2+c^2)+\frac{33bc}{2}$$

Sử dụng giả thiết : $a \geq b$, $a \geq c$. Ta có $\frac{37a(b+c)}{4} \geq \frac{37}{4}.(b^2+c^2)$

Bài toán đưa về chứng minh 

$$\frac{37}{4}(b^2+c^2)+10bc \geq 6(b^2+c^2)+\frac{33bc}{2}$$

$$\Leftrightarrow \frac{13}{4}(b-c)^2 \geq 0$$




#670214 $\sum \frac{a}{\sqrt{1+bc}}...

Đã gửi bởi royal1534 on 28-01-2017 - 10:17 trong Bất đẳng thức - Cực trị

Bài toán: Cho $a,b,c$ là các số thực dương thỏa mãn: $a^{2}+b^{2}+c^{2}=1$.

 

Chứng minh rằng: $\frac{a}{\sqrt{1+bc}}+\frac{b}{\sqrt{1+ca}}+\frac{c}{\sqrt{1+ab}}\leq \frac{3}{2}$

Áp dụng bất đẳng thức Cauchy-Schwarz ta có :

$$\sum \frac{a}{\sqrt{1+bc}} \leq \sqrt{3.(a+b+c)(\frac{a}{1+bc}+\frac{b}{1+ca}+\frac{c}{1+ab})}$$

Ta quy bài toán về chứng minh $$\frac{a}{1+bc}+\frac{b}{1+ca}+\frac{c}{1+ab} \leq \frac{9}{4(a+b+c)}$$

Ta có $$VT=(a+b+c)-abc.(\sum \frac{1}{1+bc}) \leq a+b+c-\frac{9abc}{3+ab+bc+ca} \leq a+b+c-\frac{9abc}{4}$$

Áp dụng bất đẳng thức Schur ta có :

$$9abc \geq 4(a+b+c)(ab+bc+ca)-(a+b+c)^3=(a+b+c)[(a+b+c)^2-2]=(a+b+c)^3-2(a+b+c)$$

Ta quy bài toán về chứng minh :

$$(a+b+c)^3-2(a+b+c) \leq \frac{9}{4(a+b+c)}$$

$$\Leftrightarrow 6(a+b+c)^2-(a+b+c)^4 \leq 9 \Leftrightarrow [(a+b+c)^2-3]^2 \geq 0$$

Chứng minh hoàn tất.




#667551 $x^2+y^2+z^2=p.t$

Đã gửi bởi royal1534 on 08-01-2017 - 04:40 trong Số học

Bài toán 1: Cho $p$ là số nguyên tố. Chứng minh tồn tại các số $x,y,z,t$ thỏa mãn :

$x^2+y^2+z^2=p.t$ (Với $0<t<p$)

Bài toán 2: Cho các số nguyên $a,b,c$ lớn hơn 1. Chứng minh rằng nếu với mỗi số nguyên dương $n$, tồn tại $k$ sao cho $a^k+b^k=2c^n$ thì $a=b$

Bài toán 3: Cho a,b,c là các số nguyên và $a \neq 0$ sao cho $an^2+bn+c$ là số chính phương với mọi $n>2013^{2014}$.

Chứng minh rằng tồn tại $x,y$ nguyên sao cho : $a=x^2,b=2xy,c=y^2$




#667081 Đề Thi VMO năm 2017

Đã gửi bởi royal1534 on 05-01-2017 - 14:05 trong Thi HSG Quốc gia và Quốc tế

Câu b bài hình chứng minh BP và CQ đi qua trung điểm EF
Gọi AK là đường kính của (O) và W là trung điểm BC.
Tam giác RBC và tam giác RFE đồng dạng
R, W, K thẳng hàng. Gọi T là trung điểm EF
Từ đó biến đổi góc như sau FRT=BRW=KRB=SAB=FRS nên RS đí qua T

Anh Khoa thi tốt không ạ :D 




#667012 Chứng minh : $x=y$.

Đã gửi bởi royal1534 on 05-01-2017 - 00:32 trong Số học

thì a,b,c,d thuộc Z mà bạn

Ý mình là nếu như thế thì có phải bài toán đơn giản quá không. 




#666995 Chứng minh : $x=y$.

Đã gửi bởi royal1534 on 04-01-2017 - 22:45 trong Số học

6. Cho $a,b,c,d\in Z$ thoả mãn $\frac{a^2-1}{5a}=\frac{b^2-1}{5b}=\frac{c^2-1}{4c}=\frac{d^2-1}{4d}=p$, trong đó là p nguyên dương. Chứng minh $(a-c)(b-c)(a+d)(b+d)$ là số chính phương.

Hình như đề câu này có nhầm lẫn. 

Nếu $\frac{a^2-1}{5a}$ nguyên thì $a=1,-1$ rồi chứ  :wacko: